What is System: Definition and 1000 Discussions

A system is a group of interacting or interrelated elements that act according to a set of rules to form a unified whole. A system, surrounded and influenced by its environment, is described by its boundaries, structure and purpose and expressed in its functioning. Systems are the subjects of study of systems theory.

View More On Wikipedia.org
  1. H

    Unitary vector commuting with Hamiltonian and effect on system

    Hi, I'm not sure to understand what ##| \phi_n \rangle = \sum_i \alpha_i |\psi_n^i## means exactly or how we get it. From the statement, I understand that ##[U,H] = 0## and ##H|\psi_n \rangle = E_n|\psi_n \rangle## Also, a linear combination of all states is also an solution. If U commutes...
  2. Trysse

    Geometry Looking for books (or papers) on the Cartesian coordinate system

    I am looking for more books like this one: https://archive.org/details/MethodOfCoordinateslittleMathematicsLibrary Method of Coordinaes (Little Mathematics Library) by A. S. Smogorzhevsky I am also interested in papers if you can suggest any. I am interested in texts, that explore the idea of...
  3. C

    B How do we identify a stationary system?

    In his original 1905 paper “On the Electrodynamics of Moving Bodies” Einstein's concludes his thought experiment on the definition of simultaneity by stating “It is essential to have time defined by means of stationary clocks in the stationary system, and the time now defined being appropriate...
  4. Y_G

    Ansys Maxwell - Coordinate System Limitations

    Hi everybody, I encountered a problem simulation of permanent magnets (PM) in Ansys Maxwell. There are many PM in my simulation and I need to define for each of them a proper coordinate system (CS). But I could only defined 255 CS. After that I can create new CS but It won't be possible to...
  5. H

    Mathematica How to check a particular solution of System of Linear ODEs?

    If I have been given a system of inhomogeneous linear ODEs, $$ \vec{x'} = \begin{bmatrix} 4 & -1 \\ 5 & -2 \\ \end{bmatrix} \vec{x} + \begin{bmatrix} 18e^{2t} \\ 30e^{2t}\\ \end{bmatrix} $$ I have found its particular solution to be: $$ 1/4 \begin{bmatrix} -31e^{2t} - 25e^{6t} \\ 85e^{2t} -...
  6. chris25

    Which system to apply conservation of momentum to?

    For this problem I was very confused whether conservation of angular momentum should be applied to the person, the swing or the person-swing system. It seems to me that there is no net torque on any of the three systems I listed above. However, it seems that the angular momentums of the three...
  7. N

    Proving SHM for charged spring mass system in electric field

    For part (f) Solution is I don't understand why the bit highlighted in yellow is true. Would anybody be kind enough to help.
  8. MatthewKM

    I Two Entropy scenarios on a system

    Entropy question. Take a finite number of identical atoms in a specific volume of space at a moment of time. Run two thought experiments on this system scenarios (both time independent) 1: expand the volume of space of the system instantaneously by a factor of 10. The fixed number of atoms...
  9. C

    A N-Body Simulations of Chaos in the Orbits of Trappist System Planets

    As a retired physics professor with a long experience in complex simulation software for high energy physics experiments (e.g. the LHC) I revisited last July the n-body planetary simulations which I taught in an undergraduate physics course during the Spring 2017. It was then that the...
  10. B

    I Roulette system -- which optimization is better?

    Hey, gotta do some explanation first: I assume you know how roulette works. (if you dont: ball is thrown into a pit and it can either land on red, black or zero, each having a certain likeliness to land there. you can bet on where the ball will land) let's assume unrealistically you have the...
  11. C

    Symmetry behind charged spring-mass system in Electric field

    For this problem, If we assume that x = 0 is where the spring connects to the wall, then the rest position of the mass-spring-electric field position is x = EQ/k and the max position is x = 2EQ/k. Is there a reason for the symmetry between the rest position and max position? (The symmetry...
  12. skaiser

    I Issues collimating my laser with 4f system

    I am using 2 lenses in a 4f configuration. The input is a large collimated beam (632nm). After passage through the 2 lenses the beams vertical dimension remains constant, however, the horizontal dimension get smaller, as if it is being focused only in the horizontal direction. Does anyone have...
  13. C

    I In a closed loop system with a pump, how can we control the pressure?

    How can we control the pressure of the water inside a closed loop system (chiller system for example)? Let´s say, we have a pump curve and an system resistance curve that can be modified (through opening or closing some valves) In everywhere, what I see is that the intersection of the system...
  14. BurpHa

    Static Friction Required to Keep the System from Moving (Two Boxes)

    Ok, logically, it must be that the static friction force of block A equal to the force of gravity on block B, so mass of block A is: m_A * 9.8 * 0.30 = m_B * 9.8 m_A * 2.94 = 2 * 9.8 m_A * 2.94 = 19.6 m_A \approx 6.7 kg. However, when I look at block A individually, there is one thing...
  15. James1238765

    I What is the significance of the T - V Lagrangian of a system?

    Let E be a fixed immutable quantity. E can be freely exchanged between T and V, as long as $$T + V = E$$ 1. What does the quantity $$\int_x T - V $$ signify? What is the importance of this quantity? -------------------- Let E now be the budget of a factory. E can either be spent on T or V in...
  16. M

    A Adiabatic theorem for a 3 level system

    Hello! If I have a 2 level system, with the energy splitting between the 2 levels ##\omega_{12}## and an external perturbation characterized by a frequency ##\omega_P##, if ##\omega_{12}>>\omega_P## I can use the adiabatic approximation, and assume that the initial state of the system changes...
  17. D

    I Equation of motion for a simple mechanical system

    The system is shown below. It consists of a rod of length ##L## and mass ##m_b## connecting a disk of radius ##R## and mass ##m_d## to a collar of mass ##m_c## which is in turn free to slide without friction on a vertical and rigid pole. The disk rolls without slipping on the floor. The ends...
  18. B

    A Visualizing Arbitrary Coordinate System - Example Needed

    Hi I'm wondering if someone can illustrate with an example what I bracketed in blue? I'm having a hard time visualizing how it is that the accelerations of the components are NOT necessarily equal to the components of the acceleration...Much appreciated!
  19. String theory guy

    Why Measure the Center of Mass from the Same Position After a Collision?

    For example, in the problem below, if the center of mass is chosen to be measured initially at the center of the left mass, then it must be measured from the same position after the collision. This gives an initial COM of, and finial COM of, Which gives their change in center of mass of...
  20. I

    Physics of Water in the Closed System of the Great Pyramid

    Hello, I am working on a theory for a Great Pyramid power plant and I need some help understanding if my current hypothesis is even possible and how to calculate how much water will fill the upper structure. I believe the water system to work as follows: An aqueduct delivers a steady flow of...
  21. C

    How Can I Solve a System of Equations With Complex Numbers?

    How can I solve a system of equations with complex numbers 2z+w=7i zi+w=-1 I have tried substituting z with a+bi and I have tried substituting w=7i-2z but didn't get anything useful. Edit: also, I've tried, multiplying lower eq. with -1 so that I can cancel w but I get stuck with 2z and zi and...
  22. A

    Electromagnetic linear momentum for a system of two moving charges

    When you write out the equations of motion for a system of two isolated charges, you can add both of the equations and get the increase in the particles linear momentum on one side. On the other side, you get the sum of all the forces between the particles. I understand that this sum of forces...
  23. Spector989

    System of particles, impulse and conservation of angular momentum

    So i was able to solve the angular velocity part but i don't know how to find the velocity of centre of mass . For the first part i simply conserved momentum about COM because if i consider the particles as a part of the same system as rod the collision are internal forces . I am mainly...
  24. A

    A Solving this non-holonomic system using Dirac-Bergmann theory

    I have read in some books and articles that the Dirac-Bergmann procedure to deal with constraints in phase space does not care about holonomic and Non-holonomic constraints, but I've been unable to find a single example. So, I wanted to test that assertion by solving a simple non-holonomic...
  25. paulimerci

    Find the acceleration of the system (2 blocks sliding on a table)

    In both the cases 7 kg mass accelerates towards the right because of the 50N force. The unbalanced forces in both the cases are the force of gravity due to 5kg block and force of friction. Applying Newton's second law of motion to cases 1 and 2 yields the following results for acceleration...
  26. L

    I am stuck on a calculation -- Entropy change for a compound system

    Hi, Unfortunately, I have problems with the task 4 In task 3 I got the following $$ T_f=T_ie^{\Delta S_i - c_i} $$ Then I proceeded as follows $$ \Delta S = \Delta S_1 + \Delta S_1 $$ $$ \Delta S =c_1ln(\frac{T_ie^{\Delta S_i - c_i}}{T_1})+c_2ln(\frac{T_f}{T_2})$$ $$ \Delta S...
  27. zuzelle

    How does this system work? (Four pulleys and a rope lifting a mass)

    I used the regular way of solving problems with pulleys. I tried to find the tension forces, which must be the same in the rope, but I get it wrong
  28. M

    Finding COM of L-Shaped Rod and Ball System

    I suspect we can treat the L shaped rod as two rods and then find the com for that system and then find the com of the ball and that system
  29. andris0110

    Vertical Spring System (Lagrangian)

    I am trying to solve this and get the equations of motion using the Lagrangian method. I could do all the steps but the equations (especially the third one) seems..weird. What am I doing wrong? Sorry if the equations aren't in their simplest form, they are pulled straight from Wolfram...
  30. samy4408

    Medical The impact of the sympathetic nervous system on blood vessels

    Hello, I have some trouble understanding the role of the sympathetic nervous system in vasoconstriction and vasodilatation of blood vessels, I searched and found contradicting resources, does anyone have the right answer?
  31. Astronuc

    B Ivuna meteorite from edge of solar system to Tanzania?

    Scientists find the source of one of the rarest meteorites to fall on Earth https://www.msn.com/en-us/news/technology/scientists-find-the-source-of-one-of-the-rarest-meteorites-to-fall-on-earth/ar-AA13ejQe Terrestrial modification of the Ivuna meteorite and a reassessment of the chemical...
  32. R

    A Equation system involving InverseDigamma(Digamma(a+1)-b*k)-1

    How would one go about solving this for a and b given N, E and K? n(0) + n(1) + … + n(K) = N n(0)*0 + n(1)*1 + … + n(K)*K = E Where n(k) = InverseDigamma(Digamma(a+1)-b*k)-1
  33. S

    Derive angular frequency for mass spring system

    tried writing the x position as x = Acos(wt) (ignoring the phase) so that d2x / dt2 = -w2x Substituting that into the individual motion equations would get the required result for the individual masses, but I am not sure how to combine the equations to get the reduced mass
  34. patric44

    A Spectral solution of the two dimensional Rossler chaotic system

    Hello everyone, This is a question related to a research article involving a chaotic system that required to be solved numerically. I was trying to employ one of the spectral methods to solve the two dimensional version of the so called Rossler chaotic system...
  35. B

    Checking assumptions in boundary conditions of double well system

    The idea here (as I'm told) is to use the boundary conditions to get a transcendental equation, and then that transcendental equation can be solved numerically. So I'm making a few assumptions in this problem: 1. The potential ##V(x)## is even, so the wavefunction ##\psi(x)## is either even or...
  36. E

    System of Masses - Atwood Machine

    Would anyone verify whether or not I've formulated the proper Lagrangian here for the system above (the pulleys are massless, inextensible ropes of length ##L## and ##S##): $$\mathcal{L} = T - U $$ $$ \mathcal{L} = \frac{1}{2} m_1 { \dot l_1 }^2+ \frac{1}{2} m_2 \left( { \dot l_2}- {\dot l_1 }...
  37. shivajikobardan

    Comp Sci Data sharing in traditional file system vs dbms?

    I know data is decentralized in TFS. But how does that makes data sharing difficult? We've got distributed computing for the similar purpose on different machines as well. I read a lot on this but failed to find any information regarding why it was not possible to share data in TFS as compared...
  38. G

    I Coordinate System Transformation: Lowering/Raising Indices Explained

    In《Introducing Einstein's Relativity Ed 2》on page 106"lowering the first index with the metric,then it is easy to establish,for example by using geodesic coordinates..." In 《A First Course in General Relativity - 2nd Edition》on page 159 "If we lower the index a,we get(in the locally flat...
  39. T

    System of two pulleys and three masses

    I used m1=m2=15,0kg System: FBD: Note: I believe I have solved a) correctly and am more confused about b). a) I started with drawing the FBD. Knowing that the net sum has to be zero for the system to be at rest I used the left mass to find the tension on the rope: S1 = G1 = 15kg*9,81m/s2 =...
  40. Ahmed1029

    I Constraints of a mechanical system

    I'm studying theoretical mechanics and I kind of find the notion of a "mechanical system" very slippery, especially when it comes to constraints. Take an example : I know that when a system consists of N particles and p constraints, it has 3N-p degrees of freedom; this is the definition. Then I...
  41. R

    Troubleshooting Coordinates System from Cone

    I tried using coordinates system from cone, but not got what actually want to get. Any idea from you will greatly appreciated. Thanks
  42. B

    I Would it be possible for a black hole to have a "solar system"?

    Was just wondering if there is anything fundamentally preventing a system of planets being in permanent orbit around a black hole, without ever spiralling in. Assuming that the black hole doesn't absorb any significant amount of additional mass. Of course I know it wouldn't provide any energy to...
  43. Nintend0possum

    Immune System & Environment: Can Living Near Loved Ones Make You Sick?

    Ok.. When me and my husband were dating, I lived in a state 800 miles away from him. I noticed he got sick more often than I did.. I thought it was poor immune system and genetics, because I barely ever got sick but he and his family did. When we got married and I moved to where he was.. I...
  44. sachin

    Choosing what consists of a "system" in Newton's laws of motion

    The question is solved in a single step by taking the blocks as a system and using conservation of linear momentum in the horizontal direction as there is no net force acting in the horizontal direction. Conserving the momentum we get, m x v + M x 0 = (m+M)v', so,,v' = mv/(m +M).where v' is the...
  45. B

    A Dressed states for a 3 level system

    Hello! If we have a 2 level system (I will call the states g and e for ground and excited), and a laser field (which can have any detuning relative to the spacing between g and e), it can be shown that that the total number of particles is conserved under the laser-atom interaction hamiltonian...
  46. P

    How to model the behavior of a closed loop position system?

    Hello everyone, I would need help to model a motorized system controlled in a closed loop. It is a personal project. At first, I am looking to obtain the transfer function and simulate the system on computer. In a second step (not the aim of this thread), I would like to use this simulation to...
  47. sachin

    Applying the Work-Energy theorem to a system

    I am trying to solve the given question based on energy conservation,but am stuck with the analysis of the equations. The question says find the velocity of the bigger block when the smaller block initially given a velocity v and sliding on the horizontal part of the bigger block reaches the...
  48. S

    Engineering Pulley system with collision etc....

    Hello there, can I get some help with (b) please? first of all I wanted to ask.. can is it permitted to use different systems in one exercises? like f.e. for conservation of energy to find the velocity of ##m_3## I used as system only ##m_3## but for the collision I used the 3 masses as the...
  49. E

    B Attempt to solve this system of three linear equations

    The point (1, 5) is on the curve: y=ax^2+bx+c. This point gives the linear equation: 5 = a + b + c. A second point on the curve, (2, 10) gives the linear equation 10=4a+2b+c. A student called Erika thinks that the point (2, 19) is also on the curve. 5 = a + b + c. 10=4a+2b+c 19=4a+2b+c the...
Back
Top